Art of Problem Solving

2021 AMC 12B Problems/Problem 5: Difference between revisions

Created page with "The 2021 AMC 12B will be held on February 10th, 2021. The problems will not be made public until 24 hours after that."
 
Sugar rush (talk | contribs)
No edit summary
Line 1: Line 1:
The 2021 AMC 12B will be held on February 10th, 2021. The problems will not be made public until 24 hours after that.
==Problem==
The point <math>P(a,b)</math> in the <math>xy</math>-plane is first rotated counterclockwise by <math>90\deg</math> around the point <math>(1,5)</math> and then reflected about the line <math>y = -x</math>. The image of <math>P</math> after these two transformations is at <math>(-6,3)</math>. What is <math>b - a ?</math>
 
<math>\textbf{(A)} ~1 \qquad\textbf{(B)} ~3 \qquad\textbf{(C)} ~5 \qquad\textbf{(D)} ~7 \qquad\textbf{(E)} ~9</math>
==Solution==
<math>\boxed{\textbf{(D)} ~7}</math>

Revision as of 17:41, 11 February 2021

Problem

The point $P(a,b)$ in the $xy$-plane is first rotated counterclockwise by $90\deg$ around the point $(1,5)$ and then reflected about the line $y = -x$. The image of $P$ after these two transformations is at $(-6,3)$. What is $b - a ?$

$\textbf{(A)} ~1 \qquad\textbf{(B)} ~3 \qquad\textbf{(C)} ~5 \qquad\textbf{(D)} ~7 \qquad\textbf{(E)} ~9$

Solution

$\boxed{\textbf{(D)} ~7}$